subject
Mathematics, 06.02.2020 01:44 Clayton2847

Will mark as
what is the missing step in the given proof?
a.
statement: m∠4 = m∠15
reason: substitution property of equality
b.
statement: m∠4 + m∠8 = 180°
reason: definition of supplementary angles
c.
statement: m∠4 = m∠16
reason: transitive property of equality
d.
statement: ∠15 and ∠8 are supplementary.
reason: transitive property of equality


Will mark as  what is the missing step in the given proof?  a.  statement: m∠4 =
Will mark as  what is the missing step in the given proof?  a.  statement: m∠4 =

ansver
Answers: 2

Another question on Mathematics

question
Mathematics, 21.06.2019 18:20
What are the solution(s) to the quadratic equation x2 – 25 = 0? o x = 5 and x = -5ox=25 and x = -25o x = 125 and x = -125o no real solution
Answers: 2
question
Mathematics, 21.06.2019 19:00
Amodel rocket is launched from a roof into a large field. the path of the rocket can be modeled by the equation y = -0.04x^2 + 8.6x + 4.8, where x is the horizontal distance, in meters, from the starting point on the roof and y is the height, in meters, of the rocket above the ground. how far horizontally from its starting point will the rocket land? a. 0.56 m b. 215.56 m c. 431.11 m d. 215.74 m
Answers: 1
question
Mathematics, 21.06.2019 22:00
You're locked out of your house. the only open window is on the second floor, 25 feet above the ground. there are bushes along the edge of the house, so you will need to place the ladder 10 feet from the house. what length ladder do you need to reach the window?
Answers: 3
question
Mathematics, 21.06.2019 23:00
Is there a direction u in which the rate of change of f(x,y)equals=x squared minus 3 xy plus 4 y squaredx2−3xy+4y2 at p(1,2) equals 14? give reasons for your answer. choose the correct answer below. a. no. the given rate of change is smaller than the minimum rate of change. b. no. the given rate of change is larger than the maximum rate of change. c. yes. the given rate of change is larger than the minimum rate of change and smaller than the maximum rate of change.
Answers: 2
You know the right answer?
Will mark as
what is the missing step in the given proof?
a.
statement: m∠4 =...
Questions
question
Biology, 12.02.2020 15:52
question
Computers and Technology, 12.02.2020 15:57
question
Mathematics, 12.02.2020 16:01
question
Mathematics, 12.02.2020 16:02
question
Mathematics, 12.02.2020 16:03
question
English, 12.02.2020 16:04
Questions on the website: 13722363